• 沒有找到結果。

Advanced Calculus (I)

N/A
N/A
Protected

Academic year: 2022

Share "Advanced Calculus (I)"

Copied!
57
0
0

加載中.... (立即查看全文)

全文

(1)

Advanced Calculus (I)

WEN-CHINGLIEN

Department of Mathematics National Cheng Kung University

(2)

5.1 Riemann Integral

Definition

Let a, b ∈R with a < b.

(i)

A partition of the interval [a, b] is a set of points P = {x0,x1, . . . ,xn} such that

a = x0<x1< · · · <xn =b.

(3)

5.1 Riemann Integral

Definition

Let a, b ∈R with a < b.

(i)

A partition of the interval [a, b] is a set of points P = {x0,x1, . . . ,xn} such that

a = x0<x1< · · · <xn =b.

(4)

5.1 Riemann Integral

Definition

Let a, b ∈R with a < b.

(i)

A partition of the interval [a, b] is a set of points P = {x0,x1, . . . ,xn} such that

a = x0<x1< · · · <xn =b.

(5)

5.1 Riemann Integral

Definition

Let a, b ∈R with a < b.

(i)

A partition of the interval [a, b] is a set of points P = {x0,x1, . . . ,xn} such that

a = x0<x1< · · · <xn =b.

(6)

(ii)

The norm of a partition P = {x0,x1, . . . ,xn} is the number

||P|| = max

1≤j≤n|xj − xj−1|.

(7)

Definition (ii)

The norm of a partition P = {x0,x1, . . . ,xn} is the number

||P|| = max

1≤j≤n|xj − xj−1|.

(8)

(iii)

A refinement of a partition P = {x0,x1, . . . ,xn} is a partition Q of [a,b] that satisfies Q ⊃ P. In this case we say that Q is finer than P.

(9)

Definition (iii)

A refinement of a partition P = {x0,x1, . . . ,xn} is a partition Q of [a,b] that satisfies Q ⊃ P. In this case we say that Q is finer than P.

(10)

(iii)

A refinement of a partition P = {x0,x1, . . . ,xn} is a partition Q of [a,b] that satisfies Q ⊃ P. In this case we say that Q is finer than P.

(11)

Definition

Let a, b ∈R with a < b, let P = {x0,x1, . . . ,xn} be a partition of the interval [a,b], and suppose that f : [a, b] →R is bounded.

(i)

The upper Riemann sum of f over P is the number

U(f , P) :=

n

X

j=1

Mj(f )(xj − xj−1),

where

Mj(f ) := sup

x ∈[xj−1,xj]

f (x )

(12)

Let a, b ∈R with a < b, let P = {x0,x1, . . . ,xn} be a partition of the interval [a,b], and suppose that f : [a, b] →R is bounded.

(i)

The upper Riemann sum of f over P is the number

U(f , P) :=

n

X

j=1

Mj(f )(xj − xj−1),

where

Mj(f ) := sup

x ∈[xj−1,xj]

f (x )

(13)

Definition

Let a, b ∈R with a < b, let P = {x0,x1, . . . ,xn} be a partition of the interval [a,b], and suppose that f : [a, b] →R is bounded.

(i)

The upper Riemann sum of f over P is the number

U(f , P) :=

n

X

j=1

Mj(f )(xj − xj−1),

where

Mj(f ) := sup

x ∈[xj−1,xj]

f (x )

(14)

Let a, b ∈R with a < b, let P = {x0,x1, . . . ,xn} be a partition of the interval [a,b], and suppose that f : [a, b] →R is bounded.

(i)

The upper Riemann sum of f over P is the number

U(f , P) :=

n

X

j=1

Mj(f )(xj − xj−1),

where

Mj(f ) := sup

x ∈[xj−1,xj]

f (x )

(15)

Definition (ii)

The lower Riemann sum of f over P is the number

L(f , P) :=

n

X

j=1

mj(f )(xj − xj−1)

where

mj(f ) := inf

x ∈[xj−1,xj]f (x ).

[Note: We assumed that f is bounded so that the numbers Mj(f ) and mj(f ) would exist and be finite.]

(16)

(ii)

The lower Riemann sum of f over P is the number

L(f , P) :=

n

X

j=1

mj(f )(xj − xj−1)

where

mj(f ) := inf

x ∈[xj−1,xj]f (x ).

[Note: We assumed that f is bounded so that the numbers Mj(f ) and mj(f ) would exist and be finite.]

(17)

Definition (ii)

The lower Riemann sum of f over P is the number

L(f , P) :=

n

X

j=1

mj(f )(xj − xj−1)

where

mj(f ) := inf

x ∈[xj−1,xj]f (x ).

[Note: We assumed that f is bounded so that the numbers Mj(f ) and mj(f ) would exist and be finite.]

(18)

(ii)

The lower Riemann sum of f over P is the number

L(f , P) :=

n

X

j=1

mj(f )(xj − xj−1)

where

mj(f ) := inf

x ∈[xj−1,xj]f (x ).

[Note: We assumed that f is bounded so that the numbers Mj(f ) and mj(f ) would exist and be finite.]

(19)

Lemma:

L(f , P) ≤ U(f , P)for all partitions P and all bounded functions f.

(20)

Lemma:

L(f , P) ≤ U(f , P)for all partitions P and all bounded functions f.

(21)

Lemma:

If Pand Q are any partitions of [a,b], then L(f , P) ≤ U(f , Q).

(22)

Lemma:

If Pand Q are any partitions of [a,b], then L(f , P) ≤ U(f , Q).

(23)

Definition

Let a, b ∈R with a < b. A function f : [a, b] → R is said to be (Riemann) integrable on [a,b], if and only if f is

bounded on [a,b] and for every  > 0 there is a partition P of [a,b] such that U(f , P) − L(f , P) < .

(24)

Let a, b ∈R with a < b. A function f : [a, b] → R is said to be (Riemann) integrable on [a,b], if and only if f is

bounded on [a,b] and for every  > 0 there is a partition P of [a,b] such that U(f , P) − L(f , P) < .

(25)

Theorem

Suppose that a, b ∈R with a < b. If f is continuous on the interval [a,b], then f is integrable on [a,b].

(26)

Suppose that a, b ∈R with a < b. If f is continuous on the interval [a,b], then f is integrable on [a,b].

(27)

Proof:

Let  > 0. Since f is uniformly continuous on [a,b],choose δ >0 such that

(1) |x − y | < δ implies |f (x) − f (y )| <  b − a. Let P = {x0,x1, . . . ,xn} be any partition of [a,b] that satisfies ||P|| < δ. Fix an index j and notice, by the Extreme Value Theorem, that there are points xm and xM

in [xj−1,xj]such that

f (xm) =mj(f ) and f (xM) =Mj(f ).

(28)

Proof:

Let  > 0.Since f is uniformly continuous on [a,b], choose δ >0 such that

(1) |x − y | < δ implies |f (x) − f (y )| <  b − a. Let P = {x0,x1, . . . ,xn} be any partition of [a,b] that satisfies ||P|| < δ. Fix an index j and notice, by the Extreme Value Theorem, that there are points xm and xM

in [xj−1,xj]such that

f (xm) =mj(f ) and f (xM) =Mj(f ).

(29)

Proof:

Let  > 0. Since f is uniformly continuous on [a,b],choose δ >0 such that

(1) |x − y | < δ implies |f (x) − f (y )| <  b − a. Let P = {x0,x1, . . . ,xn} be any partition of [a,b] that satisfies ||P|| < δ. Fix an index j and notice, by the Extreme Value Theorem, that there are points xm and xM

in [xj−1,xj]such that

f (xm) =mj(f ) and f (xM) =Mj(f ).

(30)

Proof:

Let  > 0. Since f is uniformly continuous on [a,b], choose δ >0 such that

(1) |x − y | < δ implies |f (x) − f (y )| <  b − a. Let P = {x0,x1, . . . ,xn} be any partition of [a,b] that satisfies ||P|| < δ. Fix an index j and notice,by the Extreme Value Theorem, that there are points xm and xM

in [xj−1,xj]such that

f (xm) =mj(f ) and f (xM) =Mj(f ).

(31)

Proof:

Let  > 0. Since f is uniformly continuous on [a,b], choose δ >0 such that

(1) |x − y | < δ implies |f (x) − f (y )| <  b − a. Let P = {x0,x1, . . . ,xn} be any partition of [a,b] that satisfies ||P|| < δ. Fix an index j and notice, by the Extreme Value Theorem,that there are points xm and xM

in [xj−1,xj]such that

f (xm) =mj(f ) and f (xM) =Mj(f ).

(32)

Proof:

Let  > 0. Since f is uniformly continuous on [a,b], choose δ >0 such that

(1) |x − y | < δ implies |f (x) − f (y )| <  b − a. Let P = {x0,x1, . . . ,xn} be any partition of [a,b] that satisfies ||P|| < δ. Fix an index j and notice,by the Extreme Value Theorem, that there are points xm and xM

in [xj−1,xj]such that

f (xm) =mj(f ) and f (xM) =Mj(f ).

(33)

Proof:

Let  > 0. Since f is uniformly continuous on [a,b], choose δ >0 such that

(1) |x − y | < δ implies |f (x) − f (y )| <  b − a. Let P = {x0,x1, . . . ,xn} be any partition of [a,b] that satisfies ||P|| < δ. Fix an index j and notice, by the Extreme Value Theorem,that there are points xm and xM

in [xj−1,xj]such that

f (xm) =mj(f ) and f (xM) =Mj(f ).

(34)

Proof:

Let  > 0. Since f is uniformly continuous on [a,b], choose δ >0 such that

(1) |x − y | < δ implies |f (x) − f (y )| <  b − a. Let P = {x0,x1, . . . ,xn} be any partition of [a,b] that satisfies ||P|| < δ. Fix an index j and notice, by the Extreme Value Theorem, that there are points xm and xM

in [xj−1,xj]such that

f (xm) =mj(f ) and f (xM) =Mj(f ).

(35)

Since ||P|| < δ, we also have |xM − xm| < δ. Hence by (1), Mj(f ) − mj(f ) < 

(b − a). In particular,

U(f , P)−L(f , P) =

n

X

j=1

(Mj(f )−mj(f ))(xj−xj−1) <  b − a

n

X

j=1

(xj−xj−1) = .

(The last step comes from telescoping.) 2

(36)

Since ||P|| < δ,we also have |xM m

Mj(f ) − mj(f ) < 

(b − a). In particular,

U(f , P)−L(f , P) =

n

X

j=1

(Mj(f )−mj(f ))(xj−xj−1) <  b − a

n

X

j=1

(xj−xj−1) = .

(The last step comes from telescoping.) 2

(37)

Since ||P|| < δ, we also have |xM − xm| < δ. Hence by (1), Mj(f ) − mj(f ) < 

(b − a). In particular,

U(f , P)−L(f , P) =

n

X

j=1

(Mj(f )−mj(f ))(xj−xj−1) <  b − a

n

X

j=1

(xj−xj−1) = .

(The last step comes from telescoping.) 2

(38)

Since ||P|| < δ, we also have |xM m

Mj(f ) − mj(f ) < 

(b − a). In particular,

U(f , P)−L(f , P) =

n

X

j=1

(Mj(f )−mj(f ))(xj−xj−1) <  b − a

n

X

j=1

(xj−xj−1) = .

(The last step comes from telescoping.) 2

(39)

Since ||P|| < δ, we also have |xM − xm| < δ. Hence by (1), Mj(f ) − mj(f ) < 

(b − a). In particular,

U(f , P)−L(f , P) =

n

X

j=1

(Mj(f )−mj(f ))(xj−xj−1) <  b − a

n

X

j=1

(xj−xj−1) = .

(The last step comes from telescoping.) 2

(40)

Since ||P|| < δ, we also have |xM m

Mj(f ) − mj(f ) < 

(b − a). In particular,

U(f , P)−L(f , P) =

n

X

j=1

(Mj(f )−mj(f ))(xj−xj−1) <  b − a

n

X

j=1

(xj−xj−1) = .

(The last step comes from telescoping.) 2

(41)

Since ||P|| < δ, we also have |xM − xm| < δ. Hence by (1), Mj(f ) − mj(f ) < 

(b − a). In particular,

U(f , P)−L(f , P) =

n

X

j=1

(Mj(f )−mj(f ))(xj−xj−1) <  b − a

n

X

j=1

(xj−xj−1) = .

(The last step comes from telescoping.) 2

(42)

Example:

The function

f (x ) =





0 0 ≤ x < 1 2

1 1

2 ≤ x ≤ 1 is integrable on [0,1].

(43)

Example:

The function

f (x ) =





0 0 ≤ x < 1 2

1 1

2 ≤ x ≤ 1 is integrable on [0,1].

(44)

Let a, b ∈R with a < b, and f : [a, b] → R be bounded.

(i)

The upper integral of f on [a,b] is the number

(U) Z b

a

f (x )dx := inf{U(f , P) : P is a partition of [a, b]}

(ii)

The lower integral of f on [a,b] is the number

(U) Z b

a

f (x )dx := sup{L(f , P) : P is a partition of [a, b]}

(45)

Definition

Let a, b ∈R with a < b, and f : [a, b] → R be bounded.

(i)

The upper integral of f on [a,b] is the number

(U) Z b

a

f (x )dx := inf{U(f , P) : P is a partition of [a, b]}

(ii)

The lower integral of f on [a,b] is the number

(U) Z b

a

f (x )dx := sup{L(f , P) : P is a partition of [a, b]}

(46)

Let a, b ∈R with a < b, and f : [a, b] → R be bounded.

(i)

The upper integral of f on [a,b] is the number

(U) Z b

a

f (x )dx := inf{U(f , P) : P is a partition of [a, b]}

(ii)

The lower integral of f on [a,b] is the number

(U) Z b

a

f (x )dx := sup{L(f , P) : P is a partition of [a, b]}

(47)

Definition

Let a, b ∈R with a < b, and f : [a, b] → R be bounded.

(i)

The upper integral of f on [a,b] is the number

(U) Z b

a

f (x )dx := inf{U(f , P) : P is a partition of [a, b]}

(ii)

The lower integral of f on [a,b] is the number

(U) Z b

a

f (x )dx := sup{L(f , P) : P is a partition of [a, b]}

(48)

Let a, b ∈R with a < b, and f : [a, b] → R be bounded.

(i)

The upper integral of f on [a,b] is the number

(U) Z b

a

f (x )dx := inf{U(f , P) : P is a partition of [a, b]}

(ii)

The lower integral of f on [a,b] is the number

(U) Z b

a

f (x )dx := sup{L(f , P) : P is a partition of [a, b]}

(49)

Definition

Let a, b ∈R with a < b, and f : [a, b] → R be bounded.

(i)

The upper integral of f on [a,b] is the number

(U) Z b

a

f (x )dx := inf{U(f , P) : P is a partition of [a, b]}

(ii)

The lower integral of f on [a,b] is the number

(U) Z b

a

f (x )dx := sup{L(f , P) : P is a partition of [a, b]}

(50)

(iii)

If the upper and lower integrals of f on [a,b] are equal, we define the integral of f on [a,b] to be the common value

Z b a

f (x )dx := (U) Z b

a

f (x )dx = (L) Z b

a

f (x )dx .

(51)

Definition (iii)

If the upper and lower integrals of f on [a,b] are equal, we define the integral of f on [a,b] to be the common value

Z b a

f (x )dx := (U) Z b

a

f (x )dx = (L) Z b

a

f (x )dx .

(52)

(iii)

If the upper and lower integrals of f on [a,b] are equal, we define the integral of f on [a,b] to be the common value

Z b a

f (x )dx := (U) Z b

a

f (x )dx = (L) Z b

a

f (x )dx .

(53)

Remark:

If f : [a, b] →R is bounded, then its upper and lower integrals exists and are infinite, and satisfy

(L) Z b

a

f (x )dx ≤ (U) Z b

a

f (x )dx .

(54)

Remark:

If f : [a, b] →R is bounded, then its upper and lower integrals exists and are infinite, and satisfy

(L) Z b

a

f (x )dx ≤ (U) Z b

a

f (x )dx .

(55)

Theorem

Let a, b ∈R with a < b, and f : [a, b] → R be bounded.

Then f is integrable on [a,b] if and only if

(L) Z b

a

f (x )dx = (U) Z b

a

f (x )dx .

(56)

Let a, b ∈R with a < b, and f : [a, b] → R be bounded.

Then f is integrable on [a,b] if and only if

(L) Z b

a

f (x )dx = (U) Z b

a

f (x )dx .

(57)

Thank you.

參考文獻

相關文件

Department of Mathematics National Cheng Kung

Department of Mathematics National Cheng Kung

Department of Mathematics National Cheng Kung

Department of Mathematics National Cheng Kung

Department of Mathematics National Cheng Kung

Department of Mathematics National Cheng Kung

Department of Mathematics National Cheng Kung

Department of Mathematics National Cheng Kung